503-938

Συντονιστής: stranton

Άβαταρ μέλους
Christos.N
Δημοσιεύσεις: 2105
Εγγραφή: Πέμ Νοέμ 26, 2009 2:28 pm
Τοποθεσία: Ίλιον

503-938

#1

Μη αναγνωσμένη δημοσίευση από Christos.N » Τρί Μάιος 27, 2014 12:16 pm

ΘΕΜΑ 503

α) Να λύσετε την ανίσωση:\displaystyle{ 
\left| {x - \frac{1}{2}} \right| < 4 
} (Μονάδες 9)
β) Να λύσετε την ανίσωση:|x + 5| \ge 3. (Μονάδες 9)
γ) Να βρείτε τις κοινές λύσεις των ανισώσεων των ερωτημάτων (α) και (β) με χρήση του άξονα των πραγματικών αριθμών και να τις γράψετε με τη μορφή διαστήματος.(Μονάδες 7)
Απαντήσεις:

α)
\displaystyle{ 
\left| {x - \frac{1}{2}} \right| < 4 \Leftrightarrow  - 4 < x - \frac{1}{2} < 4 \Leftrightarrow  - \frac{7}{2} < x < \frac{9}{2} 
}

β)
|x + 5| \ge 3 \Leftrightarrow \left\{ \begin{array}{l} 
 x + 5 \ge 3 \\  
 x + 5 \le  - 3 \\  
 \end{array} \right. \Leftrightarrow \left\{ \begin{array}{l} 
 x \ge  - 2 \\  
 x \le  - 8 \\  
 \end{array} \right.

γ)
Βρίσκουμε με παράσταση των διαστημάτων των προηγούμενων ερωτημάτων στον άξονα των πραγματικών αριθμών:
Καταγραφή.PNG
Καταγραφή.PNG (6.08 KiB) Προβλήθηκε 4075 φορές

και στην συνέχεια γράφουμε τις κοινές λύσεις με την μορφή διαστήματος x \in \left[ { - 2,\frac{9}{2}} \right)
.



ΘΕΜΑ 504
α) Αν \alpha  < 0 , να αποδειχθεί ότι: a + \frac{1}{a} \le  - 2(Μονάδες 15)
β) Αν \alpha  < 0, να αποδειχθεί ότι:\displaystyle{ 
|a| + \left| {\frac{1}{a}} \right| \ge 2 
}.(Μονάδες 10)
Απαντήσεις:

α)
a + \frac{1}{a} \le  - 2\mathop  \Leftrightarrow \limits^{a < 0} a^2  + 1 \ge  - 2a \Leftrightarrow a^2  + 2a + 1 \ge 0 \Leftrightarrow \left( {a + 1} \right)^2  \ge 0
, το οποίο ισχύει πάντα.
β)
Αν a < 0
τότε |a| =  - a
συνεπώς: \displaystyle{ 
|a| + \left| {\frac{1}{a}} \right| \ge 2\mathop  \Leftrightarrow \limits^{a < 0}  - a - \frac{1}{a} \ge 2 \Leftrightarrow a + \frac{1}{a} \le  - 2 
}
το οποίο ισχύει όπως δείξαμε στο προηγούμενο ερώτημα.
ΘΕΜΑ 505
α) Να λύσετε την εξίσωση: |2x - 4| = 3|x - 1|(Μονάδες 9)
β) Να λύσετε την ανίσωση: |3x - 5| > 1(Μονάδες 9)
γ) Είναι οι λύσεις της εξίσωσης του (α) ερωτήματος και λύσεις της ανίσωσηςτου (β) ερωτήματος; Να αιτιολογήσετε την απάντησή σας. (Μονάδες 7)
Απαντήσεις:
α)
|2x - 4| = 3|x - 1| \Leftrightarrow |2x - 4| = |3\left( {x - 1} \right)| \Leftrightarrow \left\{ \begin{array}{l} 
 2x - 4 = 3\left( {x - 1} \right) \\  
 2x - 4 =  - 3\left( {x - 1} \right) \\  
 \end{array} \right. \Leftrightarrow \left\{ \begin{array}{l} 
 2x - 4 = 3x - 3 \\  
 2x - 4 =  - 3x + 3 \\  
 \end{array} \right. \Leftrightarrow \left\{ \begin{array}{l} 
 x =  - 1 \\  
 x = \frac{7}{5} \\  
 \end{array} \right.

β)
|3x - 5| > 1 \Rightarrow \left\{ \begin{array}{l} 
 3x - 5 > 1 \\  
 3x - 5 <  - 1 \\  
 \end{array} \right. \Rightarrow \left\{ \begin{array}{l} 
 x > 2 \\  
 x < \frac{4}{3} \\  
 \end{array} \right. \Rightarrow x \in \left( { - \infty ,\frac{4}{3}} \right) \cup \left( {2, + \infty } \right)

γ)
Επειδή \frac{4}{3} = \frac{{20}}{{15}} < \frac{{21}}{{15}} = \frac{7}{5} < \frac{{10}}{5} = 2
η λύση x = \frac{7}{5}
της εξίσωσης του ερωτήματος α) δεν περιέχεται στο σύνολο των λύσεων της ανίσωσης του ερωτήματος β).
ΘΕΜΑ 506

Αν 2 \le x \le 3 και 1 \le y \le 2, να βρείτε μεταξύ ποιών ορίων βρίσκεται η τιμή καθεμιάςαπό τις παρακάτω παραστάσεις:

α) x + y (Μονάδες 5)
β) 2x - 3y (Μονάδες 10)
γ) \frac{y}{x}(Μονάδες 10)
Απαντήσεις:

α)
\left. \begin{array}{l} 
 2 \le x \le 3 \\  
 1 \le y \le 2 \\  
 \end{array} \right\}\mathop  \Rightarrow \limits^ +  3 \le x + y \le 5

β)
\left. \begin{array}{l} 
 2 \le x \le 3 \\  
 1 \le y \le 2 \\  
 \end{array} \right\} \Rightarrow \left. \begin{array}{l} 
 2 \cdot 2 \le 2x \le 2 \cdot 3 \\  
 1\left( { - 3} \right) \ge  - 3y \ge 2\left( { - 3} \right) \\  
 \end{array} \right\} \Rightarrow \left. \begin{array}{l} 
 \,\,4 \le 2x \le 6 \\  
  - 6 \le  - 3y \le  - 3 \\  
 \end{array} \right\}\mathop  \Rightarrow \limits^ +   - 2 \le 2x - 3y \le 3

γ)
\left. \begin{array}{l} 
 2 \le x \le 3 \\  
 1 \le y \le 2 \\  
 \end{array} \right\} \Rightarrow \left. \begin{array}{l} 
 2 \le x \le 3 \\  
 \frac{1}{2} \le \frac{1}{y} \le 1 \\  
 \end{array} \right\}\mathop  \Rightarrow \limits^ \times  1 \le \frac{x}{y} \le 3
ΘΕΜΑ 507

Δίνεται η εξίσωση: \left( {\lambda ^2  - 9} \right)x = \lambda ^2  - 3\lambda
, με παράμετρο \lambda  \in R(1)
α) Επιλέγοντας τρείς διαφορετικές πραγματικές τιμές για το \lambda, να γράψετε τρείς εξισώσεις.(Μονάδες 6)
β) Να προσδιορίσετε τις τιμές του\lambda  \in R, ώστε η (1) να έχει μία και μοναδική λύση.(Μονάδες 9)
γ) Να βρείτε την τιμή του \lambda  \in R, ώστε η μοναδική λύση της (1) να ισούται με 4.(Μονάδες 10)
Απαντήσεις:
α) Διαδοχικά για \lambda  = 0,1,2έχουμε:

\begin{array}{l} 
  - 9x = 0 \\  
  - 8x =  - 2 \\  
  - 5x =  - 2 \\  
 \end{array}

β) Για να έχουμε μοναδική λύση αρκεί: \lambda ^2  - 9 \ne 0 \Leftrightarrow \left( {\lambda  - 3} \right)\left( {\lambda  + 3} \right) \ne 0 \Leftrightarrow \lambda  - 3 \ne 0\,\,\kappa \alpha \iota \,\,\lambda  + 3 \ne 0 \Leftrightarrow \lambda  \ne  \pm 3

γ) Η τιμή x = 4
πρέπει να επαληθεύει την εξίσωση:
\begin{array}{l} 
 \left( {\lambda ^2  - 9} \right)4 = \lambda ^2  - 3\lambda  \Leftrightarrow 4\lambda ^2  - 36 = \lambda ^2  - 3\lambda  \Leftrightarrow 3\lambda ^2  + 3\lambda  - 36 = 0 \Leftrightarrow  \\  
  \\  
 \lambda ^2  + \lambda  - 12 = 0 \Leftrightarrow \lambda  = \frac{{ - 1 \pm 7}}{2}\mathop  \Rightarrow \limits^{\lambda  \ne  \pm 3} \lambda  =  - 4 \\  
 \end{array}
ΘΕΜΑ 508
α) Να βρείτε το άθροισμα των vπρώτων διαδοχικών θετικών ακεραίων 1,2,3, \ldots ,v (Μονάδες 12)
β) Να βρείτε πόσους από τους πρώτους διαδοχικούς θετικούς ακέραιους πρέπει ναχρησιμοποιήσουμε για να πάρουμε άθροισμα τον αριθμό 45.(Μονάδες 13)
Απαντήσεις:
α) Πρόκειται για αριθμητική πρόοδο με γενικό όρο a_vόπου a_1  = 1και διαφορά \omega  = 1
, άρα το άθροισμα βρίσκεται από τον τύπο:

\displaystyle{ 
S_v  = \frac{v}{2}\left[ {2a_1  + \left( {v - 1} \right)\omega } \right] = \frac{v}{2}\left[ {2 + v - 1} \right] = \frac{v}{2}\left( {v + 1} \right) 
}

β) Αρκεί να λύσουμε την εξίσωση:
\displaystyle{ 
\begin{array}{l} 
 S_v  = 45 \Rightarrow \frac{v}{2}\left( {v + 1} \right) = 45 \Rightarrow v\left( {v + 1} \right) = 90 \Rightarrow v^2  + v - 90 = 0 \Rightarrow v = \frac{{ - 1 \pm \sqrt {361} }}{2}\mathop  \Rightarrow \limits^{v > 0}  \\  
  \\  
 v = \frac{{\sqrt {361}  - 1}}{2} = \frac{{19 - 1}}{2} = 9 \\  
 \end{array} 
}
ΘΕΜΑ 509
α) Αν \alpha ,\beta  \in R - \{ 0\}, να αποδειχθεί ότι:\left| {\frac{\alpha }{\beta }} \right| + \left| {\frac{\beta }{\alpha }} \right| \ge 2(1)(Μονάδες 15)

β) Πότε ισχύει η ισότητα στην (1); Να αιτιολογήσετε την απάντησή σας.(Μονάδες 10)
Απαντήσεις:
α) Για \alpha ,\beta  \in R - \{ 0\}

\begin{array}{l} 
 \left| {\frac{\alpha }{\beta }} \right| + \left| {\frac{\beta }{\alpha }} \right| \ge 2 \Leftrightarrow \frac{{|\alpha |}}{{|\beta |}} + \frac{{|\beta |}}{{|\alpha |}} \ge 2 \Leftrightarrow \frac{{|\alpha |^2  + |\beta |^2 }}{{|\alpha ||\beta |}} \ge 2 \Leftrightarrow |\alpha |^2  + |\beta |^2  \ge 2|\alpha ||\beta | \Leftrightarrow  \\  
  \\  
  \Leftrightarrow |\alpha |^2  + |\beta |^2  - 2|\alpha ||\beta | \ge 0 \Leftrightarrow \left( {|\alpha | - |\beta |} \right)^2  \ge 0 \\  
 \end{array}

το παραπάνω ισχύει πάντα.

β)
\begin{array}{l} 
 \left| {\frac{\alpha }{\beta }} \right| + \left| {\frac{\beta }{\alpha }} \right| = 2 \Leftrightarrow \frac{{|\alpha |}}{{|\beta |}} + \frac{{|\beta |}}{{|\alpha |}} = 2 \Leftrightarrow \frac{{|\alpha |^2  + |\beta |^2 }}{{|\alpha ||\beta |}} = 2 \Leftrightarrow |\alpha |^2  + |\beta |^2  = 2|\alpha ||\beta | \Leftrightarrow  \\  
  \\  
  \Leftrightarrow |\alpha |^2  + |\beta |^2  - 2|\alpha ||\beta | = 0 \Leftrightarrow \left( {|\alpha | - |\beta |} \right)^2  = 0 \Leftrightarrow |\alpha | = |\beta | \Leftrightarrow \alpha  =  \pm \beta  \\  
 \end{array}


ΘΕΜΑ 510
Δίνεται η συνάρτηση f
, με: f(x) = \left\{ \begin{array}{l} 
 2x - 5,\,\,\,x \le 3 \\  
 x^2 \,,\,\,\,\,\,\,\,\,\,\,3 < x < 10 \\  
 \end{array} \right.

α) Να γράψετε το πεδίο ορισμού της συνάρτησης f σε μορφή διαστήματος.(Μονάδες 8)
β) Να υπολογίσετε τις τιμές f\left( { - 1} \right),f\left( 3 \right)και f\left( 5 \right). (Μονάδες 8)
γ) Να λύσετε την εξίσωση f\left( x \right) = 25. (Μονάδες 9)
Απαντήσεις:
α) Βρίσκουμε την ένωση των διαστημάτων που ορίζονται ο κλάδοι της συνάρτησης, έτσι έχουμε:
\left\{ \begin{array}{l} 
 x \le 3 \\  
 3 < x < 10 \\  
 \end{array} \right. \Rightarrow x < 10 \Rightarrow x \in \left( { - \infty ,10} \right)

β) Με κατάλληλη επιλογή κάθε κλάδου:
\begin{array}{l} 
 f\left( { - 1} \right) = 2\left( { - 1} \right) - 5 =  - 2 - 5 =  - 7 \\  
  \\  
 f\left( 3 \right) = 2 \cdot 3 - 5 = 6 - 5 = 1 \\  
  \\  
 f\left( 5 \right) = 5^2  = 25 \\  
 \end{array}

γ) Λύνουμε την εξίσωση για κάθε κλάδο, στην συνέχεια επαληθεύουμε τις λύσεις.
f\left( x \right) = 25\mathop  \Rightarrow \limits^{x \le 3} 2x - 5 = 25 \Rightarrow 2x = 30 \Rightarrow x = 15
η οποία απορρίπτεται.
f(x) = 25\mathop  \Rightarrow \limits^{3 < x < 10} x^2  = 25 \Rightarrow x =  \pm 5\mathop  \Rightarrow \limits^{3 < x < 10} x = 5

ΘΕΜΑ 936
Δίνεται η παράσταση: A = \left( {\sqrt {x - 4}  + \sqrt {x + 1} } \right)\left( {\sqrt {x - 4}  - \sqrt {x + 1} } \right)

α) Για ποιες τιμές του x ορίζεται η παράσταση Α; Να αιτιολογήσετε την απάντησή σας.(Μονάδες 12)
β) Να αποδείξετε ότι η παράσταση Α είναι σταθερή, δηλαδή ανεξάρτητη του x.(Μονάδες 13)
Απαντήσεις:
α) Η παράσταση ορίζεται αν :
\left\{ \begin{array}{l} 
 x - 4 \ge 0 \\  
 x + 1 \ge 0 \\  
 \end{array} \right. \Rightarrow \left\{ \begin{array}{l} 
 x \ge 4 \\  
 x \ge  - 1 \\  
 \end{array} \right.\mathop  \Rightarrow \limits^{4 >  - 1} x \ge 4
,
άρα ορίζεται για εκείνα τα x τα οποία βρίσκονται στο διάστημα [4, + \infty )
.
β)A = \left( {\sqrt {x - 4}  + \sqrt {x + 1} } \right)\left( {\sqrt {x - 4}  - \sqrt {x + 1} } \right) = \left( {\sqrt {x - 4} } \right)^2  - \left( {\sqrt {x + 1} } \right)^2  = x - 4 - x - 1 =  - 5
ΘΕΜΑ 938
α) Να δείξετε ότι: \displaystyle{ 
3 < \sqrt[3]{{30}} < 4 
} (Μονάδες 12)

β) Να συγκρίνετε τους αριθμούς \displaystyle{ 
\sqrt[3]{{30}} 
}
και \displaystyle{ 
6 - \sqrt[3]{{30}} 
} (Μονάδες 13)
Απαντήσεις:
α) Έχουμε: \displaystyle{ 
3 < \sqrt[3]{{30}} < 4 \Leftrightarrow 3^3  < \sqrt[3]{{30}}^3  < 4^3  \Leftrightarrow 27 < 30 < 64 
}
το οποίο ισχύει.
β) Από το πρώτο ερώτημα έχουμε :
\displaystyle{ 
3 < \sqrt[3]{{30}} < 4 \Leftrightarrow  - 4 <  - \sqrt[3]{{30}} <  - 3 \Leftrightarrow 6 - 4 < 6 - \sqrt[3]{{30}} < 6 - 3 \Leftrightarrow 2 < 6 - \sqrt[3]{{30}} < 3 
}

ενώ \displaystyle{ 
3 < \sqrt[3]{{30}} < 4 
}
, άρα \displaystyle{ 
2 < 6 - \sqrt[3]{{30}} < 3 < \sqrt[3]{{30}} < 4 \Rightarrow 6 - \sqrt[3]{{30}} < \sqrt[3]{{30}} 
}


Χρήστος Ντάβας
Wir müssen wissen — wir werden wissen! D.Hilbert
Άβαταρ μέλους
Καρδαμίτσης Σπύρος
Επιμελητής
Δημοσιεύσεις: 2338
Εγγραφή: Κυρ Δεκ 21, 2008 11:14 pm
Επικοινωνία:

Re: 503-938

#2

Μη αναγνωσμένη δημοσίευση από Καρδαμίτσης Σπύρος » Τρί Μάιος 27, 2014 2:07 pm

Μέχρι τώρα (Τρίτη 27 Μαίου ) έχουμε :
ΓΙΑ ΑΛΓΕΒΡΑ Α ΛΥΚΕΙΟΥ

mathxl 481-499 και η 1015
gavrilos 7677-8458
Christos.N 503 - 938
Κατσίπης 944 - 1005
Τηλέγραφος 1007-1057 (να μην λυθεί η 1015)
perpant 1062 - 1088
Στόγιας 1089 - 1101
panosG 1102- 1287
exdx 1288-1509
Χασάπης 1521 - 1533
Καναβής 2212 - 3828
Νικολόπουλος 3859 - 4299
Παγώνης 4295 – 4308
sifis 4308 – 8173
mg2002 473-480
Πρωτοπαπάς 1868 - 2055
Ιωάννου 2064 – 2229
ji2mada 2006 2234 - 2313
Μανιατοπουλου Αμαλία 2323 – 4551
Valaranko 4558-4660
Λιναρδάτος 7552 – 7974
Νικολόπουλος (β δόση) 7506-7522 και 10774, 10775
Λαζαρίδης 4828 -4861
Panpdop 6227 – 7504
Κουτσούδης 4663 - 4819
Παγώνης 4862-4965

.............................
..............................


Γράφουμε σε word: αριθμός άσκησης όπως την έδωσε το ΙΕΠ -
εκφώνηση και λύση.

1-Γραμματοσειρά TimesnewRoman
2-Μέγεθος Γραμματοσειράς 12
3-όλα τα μαθηματικά σύμβολα σε math type
4-Αναλυτικός τρόπος λύσης
5- Κάθε λύση να δημοσιεύεται και σε latex


Mόλις ετοιμαστούν τα στέλνετε στο μέηλ μου

spyroskardamitsis@hotmail.com

σε αρχείο doc (το προτιμώ γιατί έχω παλιό υπολογιστή) με τίτλο τον αριθμό των ασκήσεων πχ 503 – 938
και σε δεύτερο αρχείο γραμμένα με latex (όποιος δεν γνωρίζει τα μετατρέπω εγώ)

Αν κάποιος τελείωσε την δεκάδα του και έχει κουράγιο να συνεχίσει ευχαρίστως με πμ να τον καταγράψω και σε άλλη ομάδα ασκήσεων

Τέλος 4 δεκάδες ασκήσεων ζητούν εθελοντές και παράκληση μην αναλαμβάνεται λύση ασκήσεων χωρίς να ενημερωθώ γίνονται διπλοί κόποι


Καρδαμίτσης Σπύρος
Άβαταρ μέλους
cretanman
Διαχειριστής
Δημοσιεύσεις: 4097
Εγγραφή: Πέμ Δεκ 18, 2008 12:35 pm
Τοποθεσία: Ηράκλειο Κρήτης
Επικοινωνία:

Re: 503-938

#3

Μη αναγνωσμένη δημοσίευση από cretanman » Τρί Μάιος 27, 2014 3:00 pm

Ετοιμάζω τις ασκήσεις 4970 έως 5882 μετά από συνεννόηση με το Σπύρο Καρδαμίτση.

Αλέξανδρος


Αλέξανδρος Συγκελάκης
Απάντηση

Επιστροφή σε “Τράπεζα Θεμάτων, Άλγεβρα A”

Μέλη σε σύνδεση

Μέλη σε αυτήν τη Δ. Συζήτηση: Δεν υπάρχουν εγγεγραμμένα μέλη και 1 επισκέπτης